Solve Hydrostatic Force w/ Triangle Plate - Math Homework

In summary, a triangular plate with a height of 6 ft and a base of 8 ft is submerged vertically in water with its top 3 ft below the surface. The hydrostatic force against one side of the plate can be expressed as an integral, which is the product of the pressure and the area of the submerged surface. The width of the triangle can be found using the ratio w/8 = (9-x)/6, where x is the depth measured from the surface. The weight density of water, 62.5 lb/ft^3, represents rho*g in the equation. The integral needs to be corrected for the width and the depth coordinate, which should be x-3. The correct integral is F = \int
  • #1
stunner5000pt
1,461
2

Homework Statement


A triangular plate with height 6 ft and a base of 8 ft is submerged vertically in water so that the top is 3 ft below the surface. Express the hydrostatic force against one side of the plate as an integral and evaluate it. (Recall that the weight density of water is 62.5 lb/ft3.)


Homework Equations



hydrostatic pressure is [itex] p = \rho g h[/itex]

The Attempt at a Solution


The hydrostatic force is the pressure times the area of the surface that is submerged

the width of the triangle is solved using the following ratio:
[tex] \frac{w}{8}=\frac{9-x}{6} [/tex]

[tex] w = \frac{8}{6} (9-x) [/tex]

The force is then given by:

[tex] \int_{3}^{9} \rho g \frac{8}{6} (9-x) [/tex]

Are the upper and lower bounds of the integral correct?
In this question the weight density is given in lb/ft^3 - does this value replace the value of rho in the above equation or does it represent rho times g?

Thanks for your help!
 
Physics news on Phys.org
  • #2
The weight density of fresh water is 62.5 lb force / ft^3. This value represents rho*g.

You don't explicitly define x. Your integral has an expression for the width of the triangle (which I would check again), but it lacks dx.

I would check to see if your relation for the width gives zero width for depths from 0 to 3 feet. If this relation is in error, I suggest a change in the limits of integration to match the depths of the base and apex of the triangle.
 
  • #3
Your integral is also missing h to convert rho*g to a pressure.
 
  • #4
SteamKing said:
The weight density of fresh water is 62.5 lb force / ft^3. This value represents rho*g.

You don't explicitly define x. Your integral has an expression for the width of the triangle (which I would check again), but it lacks dx.

I would check to see if your relation for the width gives zero width for depths from 0 to 3 feet. If this relation is in error, I suggest a change in the limits of integration to match the depths of the base and apex of the triangle.

SteamKing said:
Your integral is also missing h to convert rho*g to a pressure.

Thank you for your help. Based on what you've told, I correct the integral to this:

[tex] F = \int_{3}^{9} \rho g \frac{8}{6}\left( 9 - x \right) x dx [/tex]

How does this look?
 
  • #5
Since the apex of the triangle is submerged, there can be no additional area from a depth of 3 feet to 0 feet. Your expression for the width of the triangle needs some adjustment to account for this.
 
  • #6
SteamKing said:
Since the apex of the triangle is submerged, there can be no additional area from a depth of 3 feet to 0 feet. Your expression for the width of the triangle needs some adjustment to account for this.

Does this mean that the 9 - x should be actually 6 - x?

From the question, the triangle is inverted and submerged 3 ft under water... wouldn't the force be from 0 depth to 9 ddepth??
 
Last edited:
  • #7
I think you need to take a moment and make a sketch of the problem.

The bottom of the triangle is 9 feet below the surface, and the top is 3 feet below.
From 3 feet below to the surface, there is no triangle, so the width is zero.

If x is going to be your overall depth measured from the surface, then the vertical position relative to the base of the triangle must be the depth minus 3 feet. The depth is required to determine hydrostatic pressure, but the depth coordinate must be modified when determining position between the base and the top of the triangle.
 

1. How do you calculate hydrostatic force on a triangle plate?

The hydrostatic force on a triangle plate can be calculated by multiplying the area of the plate by the pressure of the fluid at the centroid of the plate. The formula for this is F = P * A, where F is the hydrostatic force, P is the pressure, and A is the area of the plate.

2. What is the formula for finding the centroid of a triangle plate?

The formula for finding the centroid of a triangle plate is (x,y) = ((x1+x2+x3)/3, (y1+y2+y3)/3), where (x,y) represents the coordinates of the centroid and (x1,y1), (x2,y2), and (x3,y3) represent the coordinates of the vertices of the triangle.

3. How do you determine the pressure of the fluid on a triangle plate?

The pressure of the fluid on a triangle plate can be determined by using the formula P = ρgh, where ρ is the density of the fluid, g is the acceleration due to gravity, and h is the height of the fluid above the plate. This formula assumes that the fluid is incompressible and at rest.

4. Can you use the same formula for calculating hydrostatic force on any shape of plate?

Yes, the formula for calculating hydrostatic force can be used for any shape of plate as long as the centroid and area of the plate are known. The pressure used in the formula may vary depending on the shape and orientation of the plate.

5. How can I check my calculations for hydrostatic force on a triangle plate?

You can check your calculations by using the principle of moments, which states that the sum of clockwise moments about a point is equal to the sum of anticlockwise moments about the same point. In this case, the point would be the centroid of the triangle plate. If the two sums are equal, then your calculations are correct.

Similar threads

  • Engineering and Comp Sci Homework Help
Replies
1
Views
2K
  • Engineering and Comp Sci Homework Help
Replies
9
Views
2K
  • Engineering and Comp Sci Homework Help
Replies
11
Views
1K
  • Engineering and Comp Sci Homework Help
Replies
6
Views
5K
  • Engineering and Comp Sci Homework Help
Replies
2
Views
1K
  • Mechanics
Replies
4
Views
3K
  • Introductory Physics Homework Help
Replies
14
Views
1K
  • Engineering and Comp Sci Homework Help
Replies
1
Views
2K
  • Engineering and Comp Sci Homework Help
Replies
14
Views
6K
  • Introductory Physics Homework Help
Replies
1
Views
1K
Back
Top